Foro de preguntas y respuestas de Física

logo beUnicoos
Los foros de unicoos se han unificado en nuestra nueva plataforma beUnicoos. Para dejar nuevas preguntas deberás hacerlo allí, donde además podrás encontrar nuevas asignaturas y herramientas para ayudarte más con tus estudios.

  • icon

    Señor Oscuro
    el 8/12/19

    Hola, alguien podria ayudarme con este ejercicio de examen?


    replythumb_up0 voto/sflag
    icon

    Raúl RC
    el 8/12/19

    Hola, lamento no poder ayudarte pero no resolvemos dudas universitarias que no tengan que ver específicamente con los vídeos ya grabados por el profe, lo lamento de corazón. Espero algún otro unicoos universitaria se anime a poder ayudarte, de hecho lo ideal sería que os ayudárais los unos a los otros

    thumb_up0 voto/sflag
    icon

    Antonio Silvio Palmitano
    el 9/12/19

    Vamos con una orientación.

    Considera un sistema de referencia con eje OY vertical con sentido positivo hacia arriba.

    Luego, tienes que sobre la boya están aplicadas tres fuerzas verticales, de las que indicamos sus módulos y sentidos (queda para ti que hagas el diagrama de fuerzas):

    Peso: PBδB*VB*g, hacia abajo,

    Empuje del líquido: EBδL*VBS*g, hacia arriba,

    Tensión de la cuerda: T, hacia abajo;

    luego, aplicas la Primera Ley de Newton, y tienes la ecuación:

    EB - PB - T = 0, de aquí despejas:

    T = EB - PB, sustituyes las expresiones de los módulos de las fuerzas, y queda:

    T = δL*VBS*g - δB*VB*g, extraes factor común (g), y queda:

    T = (δL*VBS - δB*VB)*g (1).

    Luego, tienes que sobre el sensor están aplicadas tres fuerzas verticales, de las que indicamos sus módulos y sentidos (queda para ti que hagas el diagrama de fuerzas):

    Peso: PSe = δSe*VSe*g, hacia abajo,

    Empuje del líquido: ESe = δL*VSe*g, hacia arriba,

    Tensión de la cuerda: T, hacia arriba;

    luego, aplicas la Primera Ley de Newton, y tienes la ecuación:

    ESe - PSe + T = 0, de aquí despejas:

    T = PSe - ESe, sustituyes las expresiones de los módulos de las fuerzas, y queda:

    T = δSe*VSe*g - δL*VSe*g, extraes factores comunes (VSe, g), y queda:

    T = (δSe - δL)*VSe*g (2).

    Luego, sustituyes la expresión señalada (1) en la ecuación señalada (2), y queda:

    (δL*VBS - δB*VB)*g = (δSe - δL)*VSe*g, divides por g en ambos miembros, y queda:

    δL*VBS - δB*VB = (δSe - δL)*VSe, sumas x en ambos miembros, y queda:

    δL*VBS = δB*VB + (δSe - δL)*VSe, divides por δL*VB en todos los términos, y queda:

    VBS/VB = δB/δL + (δSe - δL)*VSe/(δL*VB),

    que es la expresión de la fracción de volumen de la boya que está sumergido, con respecto al volumen total de la boya, en función de los datos que tienes en tu enunciado;

    luego, solo queda que reemplaces valores en el segundo miembro de esta última ecuación remarcada, lo resuelvas, y luego multipliques por 100 al resultado obtenido.

    Espero haberte ayudado.

    thumb_up1 voto/sflag
  • icon

    Omar
    el 8/12/19

    Buenas. ¿Alguien me podría ayudar con el siguiente ejercicio por favor? Le he estado dando vueltas y no sé exactamente qué tengo que hallar para resolverlo.

    Gracias por adelantado :)

    replythumb_up0 voto/sflag
    icon

    Raúl RC
    el 8/12/19

    thumb_up0 voto/s

    flag
  • icon

    L E O
    el 6/12/19

    Hols, me pueden ayudar en el el siguien te ejercicio?


    tres personas quieren correr una caja hacia la derecha, la persona 1 la corre con una F1= 250N, la persona 2 lo hace con una F2= 350N y la persona 3 lo hace con F3= 500N . Realizar un diagrama de la situacion con las fuerzas involucradas, teniendo en cuenta que la escala es de 1cm = 100N.


    A)_ ¿Cual es la fuerza final con la que se corre la caja?

    B)_ ¿Que tipo de sistema corresponde? ¿Por que?

    replythumb_up0 voto/sflag
    icon

    Raúl RC
    el 8/12/19

    Simplemente has de hacer una composición de fuerzas. En este caso todas ellas van en la misma dirección y sentido, con lo cual:

    a) F1+F2+F3=250+350+500=1100 N. EL dibujo ya te lo dejo a ti

    b) En este caso te diría que son vectores concurrentes, ya que todos ellos parten de un mismo punto de aplicación. (supongo que la pregunta se refiere a eso)


    thumb_up0 voto/sflag
  • icon

    Alicia De Diego de Amorim
    el 6/12/19

    Hola, alguien me puede corregir estos ejercicios? Me estoy preparando un examen y no tengo las respuestas. Se lo agradecería muchísimo 

    replythumb_up0 voto/sflag
    icon

    Antonio Silvio Palmitano
    el 6/12/19

    1)

    Observa que debes corregir, porque el efecto mecánico de la fuerza de atracción gravitatoria que el Sol ejerce Sobre la Tierra es proporcionar la aceleración centrípeta necesaria para que ésta mantenga su órbita circular, la cuál recorre con rapidez constante, por lo tanto tienes que el trabajo mecánico que la fuerza de atracción gravitatoria realiza es igual a cero.

    2)

    Aquí considera un sistema de referencia con eje OX horizontal con sentido positivo hacia la derecha, y considera para cado caso que θ es el ángulo determinado por la fuerza y el semieje OX positivo. Luego, planteas la expresión general del trabajo realizado por la fuerza F sobre el cuerpo (observa que indicamos con F al módulo de la fuerza aplicada, y con Δx al módulo del desplazamiento del cuerpo), y queda:

    WF = F*Δx*cosθ (1).

    Luego, tienes para cada situación:

    WFaF*Δx*cos(90°) = F*Δx*0 = 0;

    WFb = F*Δx*cos(180°) = F*Δx*(-1) = -F*Δx;

    WFc = F*Δx*cos(0°) = F*Δx*1 = WFb = F*Δx;

    WFd = F*Δx*cos(θd),

    y observa que el ángulo  pertenece al segundo cuadrante, por lo que tienes que el valor de su coseno está comprendido entre -1 y 0.

    Luego, ordenas los valores de mayor a menor, y queda la secuencia:

    WFc > WFa > WFd > WFb.

    c)

    Planteas conservación de la energía mecánica para el primer disparo, y queda la ecuación:

    EPe1 = ECt1, sustituyes las expresiones de la energía potencial elástica y de la energía cinética de traslación, y queda:

    (1/2)*k*Δx12 = (1/2)*M*v12, multiplicas por 2 en ambos miembros, y queda:

    k*Δx12 = M*v12 (1).

    Planteas conservación de la energía mecánica para el segundo disparo, y queda la ecuación:

    EPe2 = ECt2, sustituyes las expresiones de la energía potencial elástica y de la energía cinética de traslación, y queda:

    (1/2)*k*Δx22 = (1/2)*M*v22, multiplicas por 2 en ambos miembros, y queda:

    k*Δx22 = M*v22 (2).

    Luego, divides miembro a miembro la ecuación señalada (1) entre la ecuación señalada (2), simplificas en ambos miembros, y queda la ecuación:

    Δx12/Δx22 = v12/v22, asocias potencias en ambos miembros, y queda:

    (Δx1/Δx2)2 = (v1/v2)2, extraes raíz cuadrada positiva en ambos miembros, y queda:

    Δx1/Δx2 = v1/v2;

    luego, sustituyes las expresiones de las compresiones del resorte que tienes en tu enunciado (Δx1 = x, Δx2 = 2*x), simplificas en el primer miembro, y queda:

    1/2 = v1/v2, y de aquí despejas:

    v2 = 2*v1.

    Espero haberte ayudado.

    thumb_up1 voto/sflag
  • icon

    NATMY
    el 6/12/19
    flag


    Es un problema  de reflexión-refracción quisiera que me expliquen por que no utiliza solo la formula general de refraccion en este ejercicio en la otra imagen está la resolución genérica.Gracias




    replythumb_up0 voto/sflag
    icon

    Raúl RC
    el 8/12/19

    Hola, lamento no poder ayudarte pero no resolvemos dudas universitarias que no tengan que ver específicamente con los vídeos ya grabados por el profe, lo lamento de corazón. Espero algún otro unicoos universitaria se anime a poder ayudarte, de hecho lo ideal sería que os ayudárais los unos a los otros

    thumb_up0 voto/sflag
  • icon

    Daniel
    el 5/12/19

    Hola, tengo un ejercicio de ondas en el que no se como hallar la fase inicial en el apartado b. Sé hacer el a y ahí me he quedado.

    Una onda armónica transversal se propaga a lo largo de una cuerda en el sentido positivo del eje X con una amplitud de 40 cm y una velocidad de 60 cm/s. La frecuencia es 1 Hz. En el instante inicial, t = 0, en x =0 la elongación es positiva y su velocidad de oscilación es de 1,2 m/s.

    a) Calcular el periodo y la longitud de onda (0,5 puntos)

    b) Calcular la fase inicial. Escribir la ecuación de la onda en unidades del S.I.

    c) Calcular el primer instante en que la elongación es máxima en x = O (O, 75 puntos)

    d) Calcular la distancia mínima de separación entre dos puntos que tienen una diferencia de fase de (pi/6) rad.

    replythumb_up0 voto/sflag
    icon

    Antonio Silvio Palmitano
    el 5/12/19

    Planteas la expresión del coeficiente angular, y queda:

    ω = 2π*f = 2π*1 = 2π rad/s,

    además tienes los datos:

    A = 40 cm = 0,4 m,

    δ = a determinar,

    con las condiciones iniciales:

    y(0,0) = a determinar y mayor que cero,

    vy(0,0) = 1,2 m/s (suponemos que su sentido es positivo).

    Luego, planteas la expresión general de la función de onda, y queda:

    y(t,x) = A*sen(k*x - ω*t + δ), reemplazas valores, y queda:

    y(t,x) = 0,4*sen(k*x - 2π*t + δ) (1);

    luego derivas con respecto al tiempo, y la expresión general de la función velocidad de oscilación queda:

    vy(t,x) = -0,8π*cos(k*x - 2π*t + δ) (2).

    Luego, tienes la condición inicial para la velocidad de oscilación de un punto cuya abscisa es la del origen de coordenadas:

    vy(0,0) = 1,2, sustituyes la expresión señalada (2) evaluada en el primer miembro, y queda:

    -0,8π*cos(δ) = 1,2, divides por -0,8π en ambos miembros, y queda:

    cos(δ) ≅ -0,4775, compones en ambos miembros con la función inversa del coseno, y tienes dos opciones:

    1°)

    δ  118,522° ≅ 0,658π rad,

    reemplazas este valor en la ecuación señalada (1), y queda:

    y(t,x) 0,4*sen(k*x - 2π*t + 0,658π) (3),

    planteas la expresión de la condición inicial para la posición del punto en estudio, y queda:

    y(0,0) > 0, sustituyes la expresión señalada (3) evaluada en el primer miembro, y queda:

    y(0,0) ≅ 0,4*sen(0,658π≅ 0,351 > 0,

    y observa que sí se cumple la condición inicial que tienes en tu enunciado;

    2°)

    δ ≅ -61,478° ≅ -0,342π rad,

    reemplazas este valor en la ecuación señalada (1), y queda:

    y(t,x)  0,4*sen(k*x - 2π*t - 0,342π) (3),

    planteas la expresión de la condición inicial para la posición del punto en estudio, y queda:

    y(0,0) > 0, sustituyes la expresión señalada (3) evaluada en el primer miembro, y queda:

    y(0,0) ≅ 0,4*sen(-0,342π≅ -0,351 < 0,

    y observa que no se cumple la condición inicial que tienes en tu enunciado.

    Espero haberte ayudado.



    thumb_up0 voto/sflag
    icon

    Daniel
    el 5/12/19

    ¿Por qué se pone el 1,2 y no 60?

    thumb_up0 voto/sflag
    icon

    Antonio Silvio Palmitano
    el 5/12/19

    Observa que la velocidad lineal de oscilación es el dato que tienes para el punto en estudio, y su valor es 1,2 m/s.

    thumb_up0 voto/sflag
  • icon

    Marga
    el 5/12/19

    Buenos días:

    En mi instituto nos han planteado el típico problema de la pelota lanzada hacia arriba, pero cuando yo lo he resuelto con la fórmula

    V2=V02+2aS, me han dicho que no es la mejor manera. En su lugar, me han dicho que debo emplear primero la fórmula x=x 0 +v 0 t+12 at 2

    para hallar el tiempo resolviendo la ecuación de segundo grado. La distancia inicial es cero, la velocidad inicial es 30 m/s y la distancia final, 10 metros. Con esta fórmula sale un primer tiempo, que es el que tarda la pelota en llegar por primera vez a los 10 m cuando sube, y un segundo tiempo que es el que le lleva volver a pasar por esa altura a la bajada. Luego ponen el primer tiempo en la fórmula "corta", la de velocidad final es igual a velocidad inicial más la aceleración por el tiempo calculado y sacan la velocidad a los 10 metros  El problema es que salen velocidades que son algo diferentes por mi método y por este, y no sé por qué, pero mi profe tampoco me lo sabe decir. Solo dice que tenemos que hacerlo así. ¿Podéis darme una pista de por qué salen cosas diferentes?¿Hay un método más correcto que otro?


    Muchas gracias.

    replythumb_up0 voto/sflag
    icon

    Antonio Silvio Palmitano
    el 5/12/19

    Establece un sistema de referencia con eje OY vertical con sentido positivo hacia arriba, con origen de coordenadas a nivel del punto de lanzamiento del móvil, y con instante inicial: ti = 0 correspondiente al lanzamiento del proyectil.

    Luego, tienes los datos iniciales:

    yi = 0 (posición inicial),

    vi = 30 m/s (velocidad inicial),

    a = -g = -10 m/s2 (aceleración);

    luego, planteas las ecuaciones de Movimiento Rectilíneo Uniformemente Variado, y queda:

    y = yi + vi*t + (1/2)*a*t2,

    v = vi + a*t,

    v2 - vi2 + 2*a*(y - yi);

    luego, reemplazas los datos iniciales, resuelves coeficientes, cancelas términos nulos, y queda:

    y = 30*t - 5*t2 (1) (ecuación de posición),

    v = 30 - 10*t (2) (ecuación de velocidad),

    v2 - 900 = -20*y (3) ecuación velocidad-posición).

    Luego, si se trata de calcular los instantes y las velocidades para la posición en estudio: y = 10 m, tienes dos opciones: la que sugiere tu docente (primera), o la que sugieres tú (segunda), a las que consideraremos por separado.

    1°)

    Reemplazas el valor de la posición en estudio en la ecuación señalada (1), y queda:

    10 = 30*t - 5*t2, divides por 5 en todos los términos, y queda:

    2 = 6*t - t2, sumas t2 y restas 10*t en ambos miembros, y queda:

    t2 - 6*t + 2 = 0, 

    que es una ecuación polinómica cuadrática, cuyas soluciones son:

    a)

    t = (6 - √[28])/2 = 3 - √(7) s ≅ 0,354 s,

    y al reemplazar este valor en la ecuación señalada (2) queda:

    v = 30 - 10*(3 - √[7]) = 10*√(7) m/s ≅ 26,458 m/s (observa que este valor es positivo),

    por lo que tienes que en el instante remarcado el móvil se desplaza hacia arriba;

    b)

    t = (6 + √[28])/2 = 3 + √(7) s ≅ 5,646 s,

    y al reemplazar este valor en la ecuación señalada (2) queda:

    v = 30 - 10*(3 + √[7]) = -10*√(7) m/s ≅ -26,458 m/s (observa que este valor es negativo),

    por lo que tienes que en el instante remarcado el móvil se desplaza hacia abajo.

    2°)

    Reemplazas el valor de la posición en estudio en la ecuación señalada (3), y queda:

    v2 - 900 = -20*10, resuelves el segundo miembro, sumas 900 en ambos miembros, y queda:

    v2 = 700, extraes raíz cuadrada en ambos miembros, y quedan dos opciones:

    a)

    v = √(700) = 10*√(7) m/s ≅ 26,458 m/s (observa que este valor es positivo),

    y al reemplazas este valor en la ecuación señalada (2) queda:

    10*√(7) = 30 - 10*t, divides por 10 en todos los términos, y queda:

    √(7) = 3 - t, y de aquí despejas:

    t = 3 - √(7) s ≅ 0,354 s,

    por lo que tienes que en el instante remarcado el móvil se desplaza hacia arriba;

    b)

    v = -√(700) = -10*√(7) m/s ≅ -26,458 m/s (observa que este valor es negativo),

    y al reemplazas este valor en la ecuación señalada (2) queda:

    -10*√(7) = 30 - 10*t, divides por 10 en todos los términos, y queda:

    -√(7) = 3 - t, y de aquí despejas:

    t = 3 + √(7) s ≅ 5,646 s,

    por lo que tienes que en el instante remarcado el móvil se desplaza hacia abajo.

    Luego, como puedes apreciar, hemos obtenido las mismas soluciones, tanto con el método que propone tu docente, como con el método que propones tú.

    Espero haberte ayudado. 

    thumb_up1 voto/sflag
    icon

    Marga
    el 9/12/19

    ¡Muchas gracias!

    thumb_up0 voto/sflag
  • icon

    jorge velazquez
    el 5/12/19

    Me pueden ayudar con este problema no lo entiendo, se los agradecería mucho 

    replythumb_up0 voto/sflag
    icon

    Raúl RC
    el 8/12/19

    Lamento no poder ayudarte Jorge, pero en unicoos no resolvemos dudas universitarias que no tengan que ver específicamente con los vídeos ya grabados por el profe.

    Afortunadamente el profe grabó hace un tiempo una serie de vídeos relacionados con isoprocesos que espero puedan ayudarte, un saludo.

    Isoprocesos

    thumb_up1 voto/sflag
  • icon

    Alissa Bouchard
    el 4/12/19

    Buenas tardes, quisiera saber cómo comenzar con el planteamiento de este problema de momento angular

    replythumb_up0 voto/sflag
    icon

    Antonio Silvio Palmitano
    el 5/12/19

    Planteas la expresión del momento de inercia del sistema (observa que está compuesto por una varilla y por dos masas puntuales, y que el eje de giros pasa por el centro de masas de la varilla), y queda:

    I0 = (1/12)*Mv*L2 + M1*(L/2)2 + M2*(L/2)2, resuelves términos, y queda:

    I0 = (1/12)*Mv*L2 + (1/4)*M1*L2 + (1/4)*M2*L2, extraes factor común ([1/12]*L2), y queda:

    I0 = (1/12)*L2*(Mv + 3*M1 + 3*M2), reemplazas datos, y queda:

    I0 = (1/12)*0,62*(1,5 + 3*0,08 + 3*0,05), resuelves, y queda:

    I0 = 0,0567 Kg*m2.

    a)

    Planteas la expresión del módulo del momento angular en función del momento de inercia y de la rapidez angular, y queda:

    L0 = I*ω, reemplazas el valor del momento de inercia y el valor de la rapidez angular, y queda:

    I0 = 0,0567*15, resuelves, y queda:

    I0 = 0,8505 Kg*m2/s.

    b)

    Planteas la expresión del momento de fuerza total del sistema con respecto al eje de giros (observa que el peso de la varilla no produce momento, debido a que su recta de acción corta al eje de giros, y observa que consideramos positivo al sentido horario que tienes señalado en tu figura), y queda:

    τ0T = -(L/2)*cosθ*P1(L/2)*cosθ*P2, sustituyes las expresiones de los módulos de los pesos, y queda:

    τ0T = -(L/2)*cosθ*M1*g + -(L/2)*cosθ*M2*g, extraes factores comunes, y queda:

    τ0T = (L/2)*cosθ*g*(-M1 + M2), reemplazas valores, y queda:

    τ0T = (0,6/2)*cosθ*9,8*(-0,08 + 0,05), resuelves el coeficiente, y queda:

    τ0T = -0,0882*cosθ,

    y observa que el signo negativo indica que el momento de fuerza total corresponde a un giro antihorario.

    Luego, aplicas la Segunda Ley de Newton para giros, y queda la ecuación:

    τ0T = I0*α, sustituyes la expresión del módulo del momento de fuerza total y el valor del momento de inercia, y queda:

    -0,0882*cosθ = 0,8505*α, divides en ambos miembros por 0,8505, y queda:

    α ≅ -0,104*cosθ,

    que es la expresión del módulo de la aceleración angular del sistema cuando la varilla determina con la dirección un ángulo θ, como muestra tu figura.

    Espero haberte ayudado.

    thumb_up0 voto/sflag